Is the Empty Set an Inductive Set?

  • Thread starter Thread starter xlu2
  • Start date Start date
Click For Summary
The discussion centers on proving that the set of natural numbers (N) is inductive using the principle of mathematical induction, which involves showing that 1 is in the set and that if x is in the set, then x+1 is also in the set. The challenge arises with the empty set (∅), where participants debate its status as an inductive set. One participant clarifies that according to a definition from MathWorld, the empty set does not qualify as inductive since it lacks successors. However, another participant finds a way to demonstrate that the empty set can be considered inductive by defining its successors in terms of set notation. The conversation highlights the nuances of defining inductive sets in mathematical contexts.
xlu2
Messages
28
Reaction score
0

Homework Statement



Use principle of Mathematical Induction,

Prove N (set of natural numbers) is inductive.
Prove ∅ is inductive

Homework Equations


Principle of Mathematical Induction

The Attempt at a Solution



For N
Let S be a subset of N
1) 1 is element of S.
2) Suppose S is inductive for some natural numbers. If x is an element of S, then x+1 is an element of S.
3) By PMI, N is inductive for every natural number n.

Is that correct?

For ∅
Let S be a subset of ∅?
I don't know how to start. Would anyone give me a hint?

Thanks!
 
Physics news on Phys.org
Which definition of inductive set are you using? I checked MathWorld, and it suggested:
nonempty partially ordered set in which every element has a successor

Clearly, ∅ does not satisfy this.
 
  • Like
Likes 1 person
CompuChip said:
Which definition of inductive set are you using? I checked MathWorld, and it suggested:


Clearly, ∅ does not satisfy this.

Thanks. I know how to prove the empty set one now. An empty set's successor is {∅} and that one's successor is {∅,{∅}}, so on. I looked that one up on WolframAlpha.
 
Question: A clock's minute hand has length 4 and its hour hand has length 3. What is the distance between the tips at the moment when it is increasing most rapidly?(Putnam Exam Question) Answer: Making assumption that both the hands moves at constant angular velocities, the answer is ## \sqrt{7} .## But don't you think this assumption is somewhat doubtful and wrong?

Similar threads

Replies
1
Views
2K
  • · Replies 7 ·
Replies
7
Views
1K
Replies
6
Views
2K
  • · Replies 7 ·
Replies
7
Views
2K
  • · Replies 3 ·
Replies
3
Views
2K
  • · Replies 5 ·
Replies
5
Views
2K
Replies
5
Views
2K
  • · Replies 1 ·
Replies
1
Views
2K
  • · Replies 3 ·
Replies
3
Views
2K
Replies
1
Views
2K